Critical Reasoning(批判性推理) Each of the critical reasoning questions is based on a short argument, a set of statements, or a plan of action. For each question, select the best answer of the choices given.
1. Which of the following most logically completes the argument below? When mercury-vapor streetlights are used in areas inhabited by insect-eating bats, the bats feed almost exclusively around the lights, because the lights attract flying insects. In Greenville, the mercury-vapor streetlights are about to be replaced with energy-saving sodium streetlights, which do not attract insects. This change is likely to result in a drop in the population of insect-eating bats in Greenville, since
A.the bats do not begin to hunt until after sundown
B.the bats are unlikely to feed on insects that do not fly
C.the highway department will be able to replace mercury-vapor streetlights with sodium streetlights within a relatively short time and without disrupting the continuity of lighting at the locations of the streetlights
D.in the absence of local concentrations of the flying insects on which bats feed, the bats expend much more energy on hunting for food, requiring much larger quantities of insects to sustain each bat
E.bats use echolocation to catch insects and therefore gain no advantage from the fact that insects flying in the vicinity of streetlights are visible at night
A B C D E
D
[解析] Argument Construction Situation In areas with mercury-vapor streetlights, any insect-eating bats feed almost exclusively around the lights, which attract flying insects. In Greenville, mercury-vapor streetlights will soon be replaced with sodium streetlights that do not attract insects. Reasoning What evidence would suggest that the change in streetlights will reduce Greenville's population of insect-eating bats? Since the sodium streetlights will not attract flying insects, the bats will probably stop focusing their feeding around Greenville's streetlights after the lights are changed. A statement providing evidence that this will make it harder for the bats to get enough food to sustain themselves would support the conclusion that the change is likely to reduce Greenville's bat population and thus would logically complete the argument. A. Insect-eating bats existed long before streetlights did, so they can probably find insects away from streetlights even if they hunt only after sundown. B. Greenville will almost certainly still have flying insects for the bats to eat after the change, even if those insects no longer gather around the streetlights. C. If anything, such a smooth transition would be less likely to disturb the bats and therefore less likely to reduce their population. D. Correct. Since there will be no local concentrations of flying insects around Greenville streetlights after the change, the bats will most likely have more trouble getting enough to eat, and that their local population will therefore fall. E. The advantage that the bats gain from mercury-vapor streetlights comes from the high concentration of insects. The fact that the bats get no additional advantage from the insects' visibility tells us nothing about what affect the change to a different type of light might have. The correct answer is D.
2. Rats injected with morphine exhibit decreased activity of the immune system, the bodily system that fights off infections. These same rats exhibited heightened blood levels of corticosteroids, chemicals secreted by the adrenal glands. Since corticosteroids can interfere with immune-system activity, scientists hypothesized that the way morphine reduces immune responses in rats is by stimulating the adrenal glands to secrete additional corticosteroids into the bloodstream. Which of the following experiments would yield the most useful results for evaluating the scientists' hypothesis?
A.Injecting morphine into rats that already have heightened blood levels of corticosteroids and then observing their new blood levels of corticosteroids
B.Testing the level of immune-system activity of rats, removing their adrenal glands, and then testing the rats' immune-system activity levels again
C.Injecting rats with corticosteroids and then observing how many of the rats contracted infections
D.Removing the adrenal glands of rats, injecting the rats with morphine, and then testing the level of the rats' immune-system responses
E.Injecting rats with a drug that stimulates immune-system activity and then observing the level of corticosteroids in their bloodstreams
A B C D E
D
[解析] Argument Evaluation Situation Rats injected with morphine exhibit decreased immune-system activity and increased levels of corticosteroids, which are secreted by the adrenal glands and can interfere with immune-system activity. Reasoning What further experiment would help determine whether morphine reduces immune responses in rats by stimulating the adrenal glands to release more corticosteroids? Contrary to the scientists' hypothesis, the experimental results might have occurred because the morphine injections directly reduced immune-system activity. Or the injections might have blocked some mechanism that reduces corticosteroid levels in the blood, even if the morphine did not stimulate the adrenal glands to produce more corticosteroids. To evaluate whether the scientists' hypothesis is more plausible than these rival hypotheses, it would be helpful to know whether similar experimental results would occur after morphine injections even if adrenal gland activity did not change. A. Morphine could stimulate the adrenal glands of rats with normal corticosteroid levels to produce more corticosteroids, whether or not it does so in rats whose corticosteroid levels are already heightened. B. Such an experiment would not involve morphine and thus would not help to determine how morphine affects immune-system activity in rats. C. Whether or not rats contract infections may not reliably indicate their levels of immune-system activity. D. Correct. If the immune system responses decreased after the morphine injections in this experiment, the hypothesis that it was by stimulation of the adrenal glands that morphine reduced immune-system activity would be undermined. But if no decrease in immune-system responses occurred, the hypothesis would be confirmed. E. Even if the mechanism by which a drug other than morphine increases immune-system activity were discovered, this discovery would not necessarily reveal the mechanism by which morphine reduces immune-system activity. The correct answer is D.
3. Curator: If our museum lends Venus to the Hart Institute for their show this spring, they will lend us their Rembrandt etchings for our print exhibition next fall. Having those etchings will increase attendance to the exhibition and hence increase revenue from our general admission fee. Museum Administrator: But Venus is our biggest attraction. Moreover the Hart's show will run for twice as long as our exhibition. So on balance the number of patrons may decrease. The point of the administrator's response to the curator is to question
A.whether getting the Rembrandt etchings from the Hart Institute is likely to increase attendance at the print exhibition
B.whether the Hart Institute's Rembrandt etchings will be appreciated by those patrons of the curator's museum for whom the museum's biggest attraction is Venus
C.whether the number of patrons attracted by the Hart Institute's Rembrandt etchings will be larger than the number of patrons who do not come in the spring because Venus is on loan
D.whether, if Venus is lent, the museum's revenue from general admission fees during the print exhibition will exceed its revenue from general admission fees during the Hart Institute's exhibition
E.whether the Hart Institute or the curator's museum will have the greater financial gain from the proposed exchange of artworks
A B C D E
C
[解析] Argument Construction Situation A curator and a museum administrator debate whether lending a particular artwork to the Hart Institute in exchange for a loan of some of the Hart Institute's artworks would increase or decrease attendance and revenue at the museum. Reasoning Which of the curator's explicit or implicit claims is the museum administrator questioning? The administrator's statements that Venus is the museum's biggest attraction and that the Hart Institute's show will run twice as long as the museum's exhibition do not directly conflict with any statement or assumption made by the curator. However, the administrator's conclusion is that on balance the number of patrons at the museum may decrease if the curator's proposal is followed. This conclusion calls into question the curator's claim that the proposal will increase revenue from the general admission fee, since that claim presupposes that on balance the proposal will increase the number of visitors to the museum. (The context suggests that the administrator is using the term patrons to mean visitors rather than donors.) A. The administrator does not dispute that the Rembrandt etchings would probably increase attendance at the print exhibition but rather suggests that this increase would be exceeded by the loss of visitors to the museum while the Hart Institute borrows Venus. B. Neither the curator nor the administrator comments on whether the patrons attracted to the Rembrandt etchings would be the same people attracted to Venus. C. Correct. The curator implicitly infers that the former number will be larger than the latter, whereas the administrator questions this by asserting that the latter number may be larger than the former. D. The administrator does not question whether the revenue during the print exhibition will exceed the revenue during the Hart Institute's exhibition, but rather whether it will exceed the loss of revenue during the Hart Institute's exhibition. E. Neither the curator nor the administrator comments on whether the museum would gain more or less from the exchange than the Hart Institute would. The correct answer is C.
4. Which of the following most logically completes the passage? Leaf beetles damage willow trees by stripping away their leaves, but a combination of parasites and predators generally keeps populations of these beetles in check. Researchers have found that severe air pollution results in reduced predator populations. The parasites, by contrast, are not adversely affected by pollution; nevertheless, the researchers' discovery probably does explain why leaf beetles cause particularly severe damage to willows in areas with severe air pollution, since
A.neither the predators nor the parasites of leaf beetles themselves attack willow trees
B.the parasites that attack leaf beetles actually tend to be more prevalent in areas with severe air pollution than they are elsewhere
C.the damage caused by leaf beetles is usually not enough to kill a willow tree outright
D.where air pollution is not especially severe, predators have much more impact on leaf-beetle populations than parasites do
E.willows often grow in areas where air pollution is especially severe
A B C D E
D
[解析] Argument Construction Situation Leaf beetles damage willow trees, but predators and parasites keep leaf beetle populations in check. Air pollution reduces populations of predators but not of parasites. Leaf beetles damage willows especially severely in areas with severe air pollution. Reasoning What would support the conclusion that air pollution's effects on the predator populations (but not on the parasite populations) explains why leaf beetles damage willows the most in areas with severe air pollution? The word since preceding the blank space at the end of the passage indicates that the space should be filled with a premise supporting the conclusion stated immediately before the since. To support this conclusion, it would help to have evidence that predators play a predominant role in keeping leaf beetle populations in check, and thus that the reduction of predator populations by air pollution could be sufficient to enable leaf beetle populations to grow and cause especially severe damage. A. The fact that neither the predators nor the parasites directly contribute to harming the trees offers no reason to conclude that a difference in how they are affected by pollution would contribute to the harm that the beetles cause to the trees. B. If the parasites are more prevalent in areas with severe air pollution, then they are more likely to keep leaf beetle populations in check in those areas, despite the reduced predator populations. Thus, the decline in predator populations would more likely be insufficient to explain why the leaf beetles cause more damage in those areas. C. This observation is irrelevant to whether the decline in predator populations explains why leaf beetles damage willow trees more severely in areas with severe air pollution. D. Correct. This indicates that predators play a predominant role in keeping leaf beetle populations in check, so, as explained above, it supports the argument's conclusion. E. This is not clearly relevant to whether the decline in predator populations explains why leaf beetles damage willow trees more severely in areas with severe air pollution. The argument's conclusion could just as easily be true regardless of whether willows grow in such polluted areas frequently or infrequently. The correct answer is D.
5. On May first, in order to reduce the number of overdue books, a children's library instituted a policy of forgiving fines and giving bookmarks to children returning all of their overdue books. On July first there were twice as many overdue books as there had been on May first, although a record number of books had been returned during the interim. Which of the following, if true, most helps to explain the apparent inconsistency in the results of the library's policy?
A.The librarians did not keep accurate records of how many children took advantage of the grace period, and some of the children returning overdue books did not return all of their overdue books.
B.Although the grace period enticed some children to return all of their overdue books, it did not convince all of the children with overdue books to return all of their books.
C.The bookmarks became popular among the children, so in order to collect the bookmarks, many children borrowed many more books than they usually did and kept them past their due date.
D.The children were allowed to borrow a maximum of five books for a two-week period, and hence each child could keep a maximum of fifteen books beyond their due date within a two-month period.
E.Although the library forgave overdue fines during the grace period, the amount previously charged the children was minimal; hence, the forgiveness of the fines did not provide enough incentive for them to return their overdue books.
A B C D E
C
[解析] Argument Construction Situation After a library started forgiving fines and giving bookmarks to children who returned all their overdue books, the number of books returned greatly increased, but so did the number of overdue books. Reasoning Why might the policy have simultaneously increased the number of overdue books and the number of books being returned?. In order to increase both these numbers, the policy must have resulted in more books being checked out, kept past their due dates, and then returned. But why would the policy have promoted that behavior? One possibility is that it rewarded the behavior. The policy involved giving children bookmarks as rewards for returning overdue books, while removing the fines that penalized the children for doing so. If the children liked the bookmarks, they might have tried to get more of them by deliberately checking books out in order to keep them past their due dates before returning them to get the bookmarks. A. Failing to keep accurate records of the number of children would not clearly increase the number of books being returned. And the policy change did not apply to children who returned only some of their overdue books. B. This suggests that the policy had limited effects, but does not help to explain why it had apparently inconsistent effects. C. Correct. This explains how the policy gave the children a motive to check out and return more books while also allowing them to keep more of the books past the due dates. D. This restriction would have limited the number of overdue books and thus would not help to explain why that number increased. E. This suggests that the policy had little effect but does not help to explain why it had apparently inconsistent effects. The correct answer is C.
6. A certain species of desert lizard digs tunnels in which to lay its eggs. The eggs must incubate inside the tunnel for several weeks before hatching, and they fail to hatch if they are disturbed at any time during this incubation period. Yet these lizards guard their tunnels for only a few days after laying their eggs. Which of the following, if true, most helps explain why there is no need for lizards to guard their tunnels for more than a few days?
A.The eggs are at risk of being disturbed only during the brief egg-laying season when many lizards are digging in a relatively small area.
B.The length of the incubation period varies somewhat from one tunnel to another.
C.Each female lizard lays from 15 to 20 eggs, only about 10 of which hatch even if the eggs are not disturbed at any time during the incubation period.
D.The temperature and humidity within the tunnels will not be suitable for the incubating eggs unless the tunnels are plugged with sand immediately after the eggs are laid.
E.The only way to disturb the eggs of this lizard species is by opening up one of the tunnels in which they are laid.
A B C D E
A
[解析] Argument Construction Situation Lizards of a certain species dig tunnels in which they lay their eggs. Although the eggs fail to hatch if disturbed during their several weeks of incubation, the lizards guard the tunnels for only a few days after laying the eggs. Reasoning What would help to explain why the lizards have to guard their tunnels for only a few days? For the lizards to survive as a species, their behaviors must ensure that enough of their eggs hatch. Thus, they must successfully prevent enough of their eggs from being disturbed in the tunnels throughout the several weeks of incubation. If guarding the tunnels for only a few days accomplishes this, then some other factor must prevent the eggs from being disturbed during the remaining weeks. Evidence of any such factor would help to explain why the lizards do not have to guard the tunnels longer. For example, to protect the eggs without guarding them, the lizards might conceal the tunnel entrances after the first few days. Or animals likely to disturb the eggs might only be present for those first days, in which case there would be nothing for the lizards to guard against thereafter. A. Correct. This suggests that the only creatures likely to disturb the eggs are other lizards of the same species digging tunnels to lay their own eggs at around the same time. If so, each lizard can safely leave its eggs unguarded after a few days because all the other lizards will have finished digging. B. Even if the incubation period varies somewhat, the passage says it always lasts several weeks. So this does not explain why the lizards have to guard the tunnels for only a few days. C. If many eggs fail to hatch even when undisturbed, that is all the more reason for the lizards to protect the remaining eggs from disturbance throughout the incubation period so that at least some will hatch. So it does not explain why the lizards guard their tunnels only for a few days. D. Whether or not immediately plugging the tunnels with sand is enough to protect the eggs, this behavior does not explain why the lizards subsequently guard the tunnels for a few days and then leave for the rest of the incubation period. E. Even if it is impossible to disturb the eggs without opening the tunnels, that does not explain why the lizards guard the tunnels for a few days and then leave for the rest of the incubation period. The correct answer is A.
7. Most banks that issue credit cards charge interest rates on credit card debt that are ten percentage points higher than the rates those banks charge for ordinary consumer loans. These banks' representatives claim the difference is fully justified, since it simply covers the difference between the costs to these banks associated with credit card debt and those associated with consumer loans. Which of the following, if true, most seriously calls into question the reasoning offered by the banks' representatives?
A.Some lenders that are not banks offer consumer loans at interest rates that are even higher than most banks charge on credit card debt.
B.Most car rental companies require that their customers provide signed credit card charge slips or security deposits.
C.Two to three percent of the selling price of every item bought with a given credit card goes to the bank that issued that credit card.
D.Most people need not use credit cards to buy everyday necessities, but could buy those necessities with cash or pay by check.
E.People who pay their credit card bills in full each month usually pay no interest on the amounts they charge.
A B C D E
C
[解析] Argument Evaluation Situation Banks that issue credit cards tend to charge interest rates on the associated debt that are ten percentage points higher than the rates associated with "ordinary" consumer loans (consumer loans that are not associated with credit cards). Representatives of these banks have offered a justification of this practice, based on a claim that this difference in interest rates "simply covers the difference" in costs, to the banks, associated with these respective types of loans (loans associated with credit cards and consumer loans that are not associated with credit cards). Reasoning What additional facts would indicate a flaw in the bank representatives' argument? Given the description of the bank representatives' argument, we may assume that, by their estimation, the costs to banks associated with credit card debt are greater than the costs associated with other consumer loans. The representatives' argument, that the difference in interest rates "simply covers" this difference in costs, may then be seen as an argument that all of the extra money that the banks collect from the higher interest rates is necessary if the banks are to cover this difference in costs. If we can find a fact whereby the ten percentage point difference is not necessary to cover the difference in costs, then we may be able to "call into question" the bank representatives' argument. A. The point of this response to the bank representatives' argument would seem to be that the relatively high interest rates on credit debt may be justified because certain other businesses charge even higher interest rates on consumer loans. Regardless of the merits of this response, it appears intended to support the argument of the representatives, whereas our task is to identify a fact that could be used to criticize the argument. B. This purported fact does not address the argument concerning the interest rates on credit-card debt. C. Correct. If two to three percent of the value of purchases made on credit cards goes to the issuing banks, then this money could be used to cover some of the difference in costs described by the bank representatives. The interest rates on credit cards could therefore be somewhat lower than they actually are, with the difference in costs nevertheless still fully covered. The difference in interest rates of ten percentage points may therefore not be necessary. D. This point might be used in support of an argument that consumers have a genuine choice as to whether to use credit cards, and that they are therefore responsible for the higher rates of interest that they pay for credit-card debt. Such an argument would seem to support the position of bank representatives. E. As with the point in option D, this point might seem to suggest that consumers bear some of the responsibility for the higher interest rates they pay, thus perhaps mitigating the responsibility of the banks. The point might thus seem to support the position of the banks' representatives. The correct answer is C.
8. Often patients with ankle fractures that are stable, and thus do not require surgery, are given follow-up x-rays because their orthopedists are concerned about possibly having misjudged the stability of the fracture. When a number of follow-up x-rays were reviewed, however, all the fractures that had initially been judged stable were found to have healed correctly. Therefore, it is a waste of money to order follow-up x-rays of ankle fractures initially judged stable. Which of the following, if true, most strengthens the argument?
A.Doctors who are general practitioners rather than orthopedists are less likely than orthopedists to judge the stability of an ankle fracture correctly.
B.Many ankle injuries for which an initial x-ray is ordered are revealed by the x-ray not to involve any fracture of the ankle.
C.X-rays of patients of many different orthopedists working in several hospitals were reviewed.
D.The healing of ankle fractures that have been surgically repaired is always checked by means of a follow-up x-ray.
E.Orthopedists routinely order follow-up x-rays for fractures of bones other than ankle bones.
A B C D E
C
[解析] Argument Evaluation Situation Often patients with ankle fractures that their orthopedists have judged not to require surgery are given follow-up x-rays to check whether the fracture healed correctly. An examination of a sample of those x-rays found that the ankle had, in each case, healed properly. Reasoning The question is which of the options, if true, would most strengthen the argument. The argument is based on data concerning follow-up x-rays, each of which revealed no problem with the orthopedist's initial judgment that the ankle fracture was stable (and would heal without surgery). This invites the question whether the follow-up x-rays are really needed. The argument concludes that they are a waste of money. But was the x-ray data truly representative of orthopedists generally? After all, some orthopedists—perhaps more experienced, better-trained, or employed at a facility with better staff or facilities—may be much better than others at judging ankle fractures. If we add the information that the data for the conclusion comes from many orthopedists working at many different hospitals, we have greater assurance that the x-ray data is representative, and the argument will be made much stronger. A. Neither the study nor the conclusion that is drawn from it concerns general practitioners, so this point is irrelevant. B. Naturally many ankle injuries do not involve fractures—x-rays may sometimes be used to determine this—but the argument concerns only cases where there have been ankle fractures. C. Correct. This shows that the sample of x-ray data examined was probably sufficiently representative of cases of ankle fracture judged to be stable by orthopedists. D. The argument does not concern cases of ankle fracture that have been surgically repaired. E. The argument concerns only x-rays of ankles. From the information given here, we cannot infer that orthopedists are generally wasteful in routinely ordering follow-up x-rays. The correct answer is C.
9. In setting environmental standards for industry and others to meet, it is inadvisable to require the best results that state-of-the-art technology can achieve. Current technology is able to detect and eliminate even extremely minute amounts of contaminants, but at a cost that is exorbitant relative to the improvement achieved. So it would be reasonable instead to set standards by taking into account all of the current and future risks involved. The argument given concerning the reasonable way to set standards presupposes that
A.industry currently meets the standards that have been set by environmental authorities
B.there are effective ways to take into account all of the relevant risks posed by allowing different levels of contaminants
C.the only contaminants worth measuring are generated by industry
D.it is not costly to prevent large amounts of contaminants from entering the environment
E.minute amounts of some contaminants can be poisonous
A B C D E
B
[解析] Argument Construction Situation State-of-the-art technology can detect and eliminate even tiny amounts of environmental contaminants, but at a cost that is exorbitant relative to its benefits. Reasoning What must be true in order for the argument's premises to support its conclusion? The argument is that environmental standards requiring the best results that state-of-the-art technology can provide are unreasonably expensive relative to their benefits, so it would be reasonable instead to set environmental standards that take into account all present and future risks from contaminants. In order for the premise to support the conclusion, the environmental standards based on present and future risks would have to be less expensive relative to their benefits than the best results environmental standards are. Furthermore, setting the current and future risks environmental standards cannot be reasonable unless it is feasible to assess present and future risks as those standards require. A. The argument does not say which standards, if any, environmental authorities have set. In any case, such standards could be reasonable or unreasonable regardless of whether industry currently meets them. B. Correct. If taking future risks into account were infeasible, then applying the current and future risks standards would also be infeasible. And setting those standards would be unreasonable if they could not feasibly be applied. C. According to the stimulus, the proposed current and future risks standards would apply to industry and others. So those standards could be reasonable even if the unspecified others also generated contaminants worth measuring, and even if the standards required measuring those contaminants. D. Even if it were costly to prevent large amounts of contaminants from entering the environment, the benefits of doing so to prevent present and future risks might outweigh the costs. E. The current and future risks standards could take into account any poisoning risks posed by minute amounts of contaminants. The correct answer is B.
10. The chemical adenosine is released by brain cells when those cells are active. Adenosine then binds to more and more sites on cells in certain areas of the brain, as the total amount released gradually increases during wakefulness. During sleep, the number of sites to which adenosine is bound decreases. Some researchers have hypothesized that it is the cumulative binding of adenosine to a large number of sites that causes the onset of sleep. Which of the following, if true, provides the most support for the researchers' hypothesis?
A.Even after long periods of sleep when adenosine is at its lowest concentration in the brain, the number of brain cells bound with adenosine remains very large.
B.Caffeine, which has the effect of making people remain wakeful, is known to interfere with the binding of adenosine to sites on brain cells.
C.Besides binding to sites in the brain, adenosine is known to be involved in biochemical reactions throughout the body.
D.Some areas of the brain that are relatively inactive nonetheless release some adenosine.
E.Stress resulting from a dangerous situation can preserve wakefulness even when brain levels of bound adenosine are high.
A B C D E
B
[解析] Argument Evaluation Situation Adenosine is released from brain cells that are active. The amount of adenosine released increases during wakefulness, and it binds to more and more sites on cells in certain brain locations. The number of sites to which it is bound decreases during sleep. Researchers have hypothesized that the cumulative binding of adenosine to many sites causes the onset of sleep. Reasoning Which of the five pieces of information most strongly supports the hypothesis? If the hypothesis is correct, then some factor that impedes the binding of adenosine should be closely associated with wakefulness. Therefore, finding some such factor, and observing that it is accompanied by wakefulness when the factor operates, would tend to confirm the hypothesis. A. Without further, more specific information, this piece of information suffices neither to confirm nor to refute the hypothesis. B. Correct. A finding that caffeine, known to induce wakefulness, inhibits adenosine from binding to sites on brain cells helps confirm the hypothesis. C. This piece of information lacks a dear relevance to the hypothesized impact on sleep, and therefore does not help confirm the hypothesis. D. This information lacks a clear relevance to the hypothesized impact on sleep, and therefore does not help confirm the hypothesis. E. What this indicates is that stress may impede the hypothesized sleep-inducing effect of adenosine. It does not refute the hypothesis but does not confirm it either. The correct answer is B.
11. A two-year study beginning in 1977 found that, among 85-year-old people, those whose immune systems were weakest were twice as likely to die within two years as others in the study. The cause of their deaths, however, was more often heart disease, against which the immune system does not protect, than cancer or infections, which are attacked by the immune system. Which of the following, if true, would offer the best prospects for explaining deaths in which weakness of the immune system, though present, played no causal role?
A.There were twice as many infections among those in the study with the weakest immune systems as among those with the strongest immune systems.
B.The majority of those in the study with the strongest immune systems died from infection or cancer by 1987.
C.Some of the drugs that had been used to treat the symptoms of heart disease had a side effect of weakening the immune system.
D.Most of those in the study who survived beyond the two-year period had recovered from a serious infection sometime prior to 1978.
E.Those in the study who survived into the 1980s had, in 1976, strengthened their immune systems through drug therapy.
A B C D E
C
[解析] Argument Construction Situation This question presents a puzzling scenario and asks us to find a possible fact that could make the situation less puzzling. The scenario involves a study that was conducted a few decades ago on a certain group of older adults. Those with the weakest immune systems were much more likely to die within two years than were the other individuals in the study. However, among the individuals with the weakest immune systems, death was more often by heart disease, from which the immune system does not protect, than from cancer or infections, for which a strong immune system is protective. Reasoning For the participants in the study with the weakest immune systems, what might best explain the deaths that were not due to weakness of the immune system? We might expect that the people with the weakest immune systems would be more likely to die from diseases that a strong immune system would protect them from than from other diseases. An explanation of the deaths that were not due to weakness of the immune system would explain why this is not the case. A. This point is irrelevant. The hypothesis that the participants in the study with the weakest immune systems had more infections than did the other participants does not explain why those participants died from conditions that were not infections. B. Our question involves identifying a possible explanation for the deaths of the participants in the study with the weakest immune systems. This option, about the deaths of those with strong immune systems, is thus irrelevant. C. Correct. This option suggests that those with heart disease—which would not have been due to weakness of the immune system—would have nevertheless had a weaker immune system due to the administration of certain drugs. Those with heart disease may for this reason have been among those with the weakest immune systems. If the individuals with weak immune systems due to treatment for heart disease formed a large-enough portion of the patients with the weakest immune systems, then we would have an explanation for why those with the weakest immune systems were more likely to die from heart disease than from infections or cancer. D. This option is not specific enough for us to use in the explanation we are looking for. For example, the "serious" infections in question may have occurred well before the 1977 study. Furthermore, there may appear to be no significant relationship between having had a serious infection and death from a condition that was not an infection. E. This option is also not specific enough to be a factor that might reasonably offer the explanation we are looking for. For example, given the information in this option, it could have been the case that all of the participants had the drug therapy. The correct answer is C.
12. Most scholars agree that King Alfred (A.D. 849-899) personally translated a number of Latin texts into Old English. One historian contends that Alfred also personally penned his own law code, arguing that the numerous differences between the language of the law code and Alfred's translations of Latin texts are outweighed by the even more numerous similarities. Linguistic similarities, however, are what one expects in texts from the same language, the same time, and the same region. Apart from Alfred's surviving translations and law code, there are only two other extant works from the same dialect and milieu, so it is risky to assume here that linguistic similarities point to common authorship. The passage above proceeds by
A.providing examples that underscore another argument's conclusion
B.questioning the plausibility of an assumption on which another argument depends
C.showing that a principle if generally applied would have anomalous consequences
D.showing that the premises of another argument are mutually inconsistent
E.using argument by analogy to undermine a principle implicit in another argument
A B C D E
B
[解析] Argument Evaluation Situation A historian argues that King Alfred must have written his own law code, since there are more similarities than differences between the language in the law code and that in Alfred's translations of Latin texts. Apart from Alfred's translations and law code, there are only two other extant works in the same dialect and from the same milieu. Reasoning How does the reasoning in the passage proceed? The first sentence presents a claim that is not disputed in the passage. The second sentence presents a historian's argument. Implicitly citing the undisputed claim in the passage's first sentence as evidence, the historian proposes an analogy between the law code and Alfred's translations, arguing on the basis of this analogy that Alfred wrote the law code. The third sentence of the passage casts doubt on this analogy, pointing out that it could plausibly apply to texts that Alfred did not write. The fourth sentence suggests that too few extant texts are available as evidence to rule out the possibility raised in the third sentence. Thus, the third and fourth sentences are intended to undermine the historian's argument. A. As explained above, the passage is intended to undermine the conclusion of the historian's argument, not to underscore (emphasize) it. B. Correct. The passage's third and fourth sentences question the plausibility of the historian's assumption that no one but Alfred would have been likely to write a text whose language has more similarities to than differences from the language in Alfred's translations. C. Although there might well be anomalous consequences from generalizing the assumption on which the historian's argument relies, the passage does not mention or allude to any such consequences. D. The passage does not mention, or suggest the existence of, any inconsistencies among the premises of the historian's argument. E. Although the historian argues by analogy, the passage does not itself argue by analogy; it does not suggest any specific counteranalogy to undermine the historian's argument. The correct answer is B.
13. Parland's alligator population has been declining in recent years, primarily because of hunting. Alligators prey heavily on a species of freshwater fish that is highly valued as food by Parlanders, who had hoped that the decline in the alligator population would lead to an increase in the numbers of these fish available for human consumption. Yet the population of this fish species has also declined, even though the annual number caught for human consumption has not increased. Which of the following, if true, most helps to explain the decline in the population of the fish species?
A.The decline in the alligator population has meant that fishers can work in some parts of lakes and rivers that were formerly too dangerous.
B.Over the last few years, Parland's commercial fishing enterprises have increased the number of fishing boats they use.
C.Many Parlanders who hunt alligators do so because of the high market price of alligator skins, not because of the threat alligators pose to the fish population.
D.During Parland's dry season, holes dug by alligators remain filled with water long enough to provide a safe place for the eggs of this fish species to hatch.
E.In several neighboring countries through which Parland's rivers also flow, alligators are at risk of extinction as a result of extensive hunting.
A B C D E
D
[解析] Argument Construction Situation The alligators in a certain region prey heavily on a certain species offish that is prized for human consumption. However, although in recent years hunting has reduced the population of alligators in the region, the population of the prized freshwater fish species has declined. The annual number caught for human consumption has not increased. Reasoning What might explain the decline in the population of the prized fish species, despite both the decrease in population of another species that preys heavily on the prized fish and the lack of increase in fishing for the species for human consumption? The population of the fish species declined, despite both the presence of a factor that we might be expected to produce an increase in the population of the species and the absence of a factor that we might ordinarily expect to explain the decrease. This situation may seem puzzling, and we may thus wish to find an explanation for it. A. Given that fishers can work in parts of lakes and rivers that were formerly too dangerous to work in, we might expect fishing of the prized species to increase and thus expect the population of the species to decrease. Although this might explain a decrease in the population of the fish species if fishing for the species increased, we have been given reason to believe that fishing for the species decreased. B. As with option A, the statement in this option suggests that fishing in the region may have increased and thus that fishing for the prized fish species for human consumption may have increased. This might explain the decrease in the population of the fish species if the statement were correct. However, we have been given that fishing for the prized fish species for human consumption has decreased. C. The statement in this option provides an explanation of why the alligator hunting has occurred. Given that the alligators prey on the fish, this might help to explain an increase in the population of the prized fish species, had such an increase occurred. However, we are given that the population of the fish species in the region has decreased. D. Correct. Despite the fact that alligators prey on the prized fish species, this statement describes a way in which the fish species may be dependent on the alligators, in such a way that a decline in the population of the alligators could contribute to a decline in the fish species. E. The statement in this option serves to amplify a point that is given in the puzzling situation of a decline in the population of the fish species despite (among other factors) a decrease in the population of the alligators. It does not explain why a decline in the population of the alligator species may have contributed to a decline in the population of the fish species. The correct answer is D.
14. A company plans to develop a prototype weeding machine that uses curing blades with optical sensors and microprocessors that distinguish weeds from crop plants by differences in shade of color. The inventor of the machine claims that it will reduce labor costs by virtually eliminating the need for manual weeding. Which of the following is a consideration in favor of the company's implementing its plan to develop the prototype?
A.There is a considerable degree of variation in shade of color between weeds of different species.
B.The shade of color of some plants tends to change appreciably over the course of their growing season.
C.When crops are weeded manually, overall size and leaf shape are taken into account in distinguishing crop plants from weeds.
D.Selection and genetic manipulation allow plants of virtually any species to be economically bred to have a distinctive shade of color without altering their other characteristics.
E.Farm laborers who are responsible for the manual weeding of crops carry out other agricultural duties at times in the growing season when extensive weeding is not necessary.
A B C D E
D
[解析] Evaluation of a Plan Situation A company plans to develop an automated weeding machine that would distinguish weeds from crop plants by differences in shade of color. It is supposed to reduce labor costs by eliminating the need for manual weeding. Reasoning Which option describes a consideration that would favor the company's plan? The passage supports the plan by claiming that the machine would reduce labor costs by virtually eliminating weeding by hand. The correct option will be one that adds to this support. Labor costs will be reduced only if the machine works well. The machine relies on shade of color to distinguish between weeds and crop plants. If crop plants can be bred to have distinctive color without sacrificing other qualities, it would be more likely that the machine could be used effectively. A. Greater variation among weed plants would make it more difficult for the machine to distinguish between weeds and crop plants, and this would make it less likely that the machine would be effective. B. This option tends to disfavor the effectiveness of the machine. The more changeable the colors of the plants to be distinguished, the more complex the task of distinguishing between weeds and crop plants based on their color. C. This option tends to disfavor the likely benefits of the machine because it indicates that manual weeding distinguishes weeds from crop plants by using criteria that the machine does not take into account. If the machine does not distinguish weeds from crop plants as accurately and reliably as manual weeding does, then the machine is less apt to make manual weeding unnecessary. D. Correct. Making crop plants easily distinguishable from weeds would facilitate the effective use of the weeding machine. E. This does not favor the company's implementing the plan to develop the machine. There would still be tasks other than weeding that would require hiring staff. Thus there would still be labor costs even if the need for manual weeding were eliminated. The correct answer is D.
15. Aroca City currently funds its public schools through taxes on property. In place of this system, the city plans to introduce a sales tax of 3 percent on all retail sales in the city. Critics protest that 3 percent of current retail sales falls short of the amount raised for schools by property taxes. The critics are correct on this point. fNevertheless, implementing the plan will probably not reduce the money going to Aroca's schools. Several large retailers have selected Aroca City as the site for huge new stores, and these are certain to draw large numbers of shoppers from neighboring municipalities, where sales are taxed at rates of 6 percent and more. In consequence, retail sales in Aroca City are bound to increase substantially. In the argument given, the two portions in boldface play which of the following roles?
A.The first presents a plan that the argument concludes is unlikely to achieve its goal; the second expresses that conclusion.
B.The first presents a plan that the argument concludes is unlikely to achieve its goal; the second presents evidence in support of that conclusion.
C.The first presents a plan that the argument contends is the best available; the second is a conclusion drawn by the argument to justify that contention.
D.The first presents a plan one of whose consequences is at issue in the argument; the second is the argument's conclusion about that consequence.
E.The first presents a plan that the argument seeks to defend against a certain criticism; the second is that criticism.
A B C D E
D
[解析] Argument Evaluation Situation Aroca City plans to switch the source of its public school funding from property taxes to a new local sales tax. Reasoning What argumentative roles do the two portions in boldface play in the passage? The first boldface portion simply describes the city's plan. The next two sentences in the passage describe an observation some critics have made in objecting to the plan and say that the observation is correct. But then the second boldface portion rejects the critics' implicit conclusion that the plan will reduce school funding. The final two sentences in the passage present reasons to accept the statement in the second boldface portion, so they are premises supporting it as a conclusion. A. The argument concludes that the plan is unlikely to reduce funding for the schools. The passage does not mention the plan's goal, but presumably that goal is not to reduce school funding. B. The second boldface portion presents the argument's conclusion, not evidence to support the conclusion. The passage does not mention the plan's goal, but presumably that goal is not to reduce school funding. C. The passage does not say whether the plan is better than any other possible school funding plans. D. Correct. The plan's likely effect on the amount of school funding is at issue in the argument, whose conclusion is that the plan probably will not reduce that funding. E. The second boldface portion does not criticize the plan, but rather rejects a criticism of the plan by stating that the plan will probably not reduce school funding. The correct answer is D.
16. Economist: Paying extra for fair-trade coffee—coffee labeled with the Fairtrade logo—is intended to help poor farmers, because they receive a higher price for the fair-trade coffee they grow. But this practice may hurt more farmers in developing nations than it helps. By raising average prices for coffee, it encourages more coffee to be produced than consumers want to buy. This lowers prices for non-fair-trade coffee and thus lowers profits for non-fair-trade coffee farmers. To evaluate the strength of the economist's argument, it would be most helpful to know which of the following?
A.Whether there is a way of alleviating the impact of the increased average prices for coffee on non-fair-trade coffee farmers' profits
B.What proportion of coffee farmers in developing nations produce fair-trade coffee
C.Whether many coffee farmers in developing nations also derive income from other kinds of farming
D.Whether consumers should pay extra for fair-trade coffee if doing so lowers profits for non-fair-trade coffee farmers
E.How fair-trade coffee farmers in developing nations could be helped without lowering profits for non-fair-trade coffee farmers
A B C D E
B
[解析] Argument Evaluation Situation Poor farmers receive higher prices for fair-trade coffee. But paying extra for fair-trade coffee lowers prices for non-fair-trade coffee and thus lowers profits for non-fair-trade coffee farmers. Reasoning What would be most helpful to know to evaluate how well the economist's observations support the conclusion that buying fair-trade cosec hurts more farmers in developing nations than it helps? The economist suggests that buying fair-trade coffee benefits farmers who grow it because they receive higher prices, but that it hurts non-fair-trade coffee farmers by reducing their profits. So to know whether the practice hurts more farmers in developing nations than it helps, it would be helpful to know whether developing nations have more farmers who produce non-fair-trade coffee than produce fair-trade coffee. A. Even if there were some potential way of alleviating the negative impact from buying fair-trade coffee on non-fair-trade coffee farmers, it still could be that the practice hurts more developing-nation farmers than it helps. Alleviating the negative impact does not entail that there is no negative impact. B. Correct. If fewer than half of these farmers produce fair-trade coffee, then the economist's observations do suggest that buying fair-trade coffee hurts more coffee farmers in developing nations than it helps. But if more than half do, those observations suggest the contrary. C. Although knowing this could be helpful in determining how intensely many farmers are economically affected by people buying fair-trade coffee, it is not helpful in determining whether more farmers are hurt than are helped. D. The argument's conclusion is only about the economic impact of buying fair-trade coffee, not about how consumers should or should not respond to that impact. E. Knowing how the fair-trade coffee farmers could potentially be helped without hurting the other coffee farmers is irrelevant to assessing whether the practice of buying fair-trade coffee hurts more developing-nation farmers than it helps. The correct answer is B.
17. Tanco, a leather manufacturer, uses large quantities of common salt to preserve animal hides. New environmental regulations have significantly increased the cost of disposing of salt water that results from this use, and, in consequence, Tanco is considering a plan to use potassium chloride in place of common salt. Research has shown that Tanco could reprocess the by-product of potassium chloride use to yield a crop fertilizer, leaving a relatively small volume of waste for disposal. In determining the impact on company profits of using potassium chloride in place of common salt, it would be important for Tanco to research all of the following EXCEPT:
A.What difference, if any, is there between the cost of the common salt needed to preserve a given quantity of animal hides and the cost of the potassium chloride needed to preserve the same quantity of hides?
B.To what extent is the equipment involved in preserving animal hides using common salt suitable for preserving animal hides using potassium chloride?
C.What environmental regulations, if any, constrain the disposal of the waste generated in reprocessing the by-product of potassium chloride?
D.How closely does leather that results when common salt is used to preserve hides resemble that which results when potassium chloride is used?
E.Are the chemical properties that make potassium chloride an effective means for preserving animal hides the same as those that make common salt an effective means for doing so?
A B C D E
E
[解析] Evaluation of a Plan Situation New environmental regulations will increase the costs of disposing of the salt water that results from the use of large amounts of common salt in leather manufacturing. The manufacturer is considering switching from common salt to potassium chloride, because the by-product of the latter could be reprocessed to yield a crop fertilizer, with little waste left over to be disposed. Reasoning In order to determine whether it would be profitable to switch from using common salt to using potassium chloride, which of the five questions does the manufacturer NOT need to answer? The chemical properties making potassium chloride an effective means of preserving animal hides might be quite different from those that make common salt effective, but there is no particular reason for thinking that this would impact the profitability of switching to potassium chloride. The relevant effects on the preserved hides might be the same even if the properties that brought about those effects were quite different. THUS, without more information than is provided in the passage, this question is irrelevant. A. The savings in waste disposal costs that would be gained by switching to potassium chloride could be cancelled out if the cost of potassium chloride needed far exceeded that for common salt. B. If switching to potassium chloride would force the manufacturer to replace the equipment it uses for preserving hides, then it might be less profitable to switch. C. Even though there is said to be relatively little waste associated with using potassium chloride in the process, if the costs of this disposal are very high due to environmental regulations, it might be less profitable to switch. D. If the leather that results from the use of potassium chloride looks substantially different from that which results when common salt has been used, then the leather might be less attractive to consumers, which would adversely affect the economics of switching to potassium chloride. E. Correct. Note that the question as stated here presupposes that potassium chloride and salt are both effective means for preserving animal hides—so it does not raise any issue as to whether potassium chloride is adequately effective or as effective as salt (dearly, an issue of effectiveness would be relevant to profitability). The correct answer is E.
18. Colorless diamonds can command high prices as gemstones. A type of less valuable diamonds can be treated to remove all color. Only sophisticated tests can distinguish such treated diamonds from naturally colorless ones. However, only 2 percent of diamonds mined are of the colored type that can be successfully treated, and many of those are of insufficient quality to make the treatment worthwhile. Surely, therefore, the vast majority of colorless diamonds sold by jewelers are naturally colorless. A serious flaw in the reasoning of the argument is that
A.comparisons between the price diamonds command as gemstones and their value for other uses are omitted
B.information about the rarity of treated diamonds is not combined with information about the rarity of naturally colorless, gemstone diamonds
C.the possibility that colored diamonds might be used as gemstones, even without having been treated, is ignored
D.the currently available method for making colorless diamonds from colored ones is treated as though it were the only possible method for doing so
E.the difficulty that a customer of a jeweler would have in distinguishing a naturally colorless diamond from a treated one is not taken into account
A B C D E
B
[解析] Argument Evaluation Situation Colored diamonds of a type that comprises 2 percent of all mined diamonds can be treated so that they are not easily distinguishable from more valuable, naturally colorless diamonds, but many are too low in quality for the treatment to be worthwhile. Reasoning Why do the argument's premises not justify the conclusion that the vast majority of colorless diamonds sold by jewelers are naturally colorless? Since the type of colored diamonds that can be treated make up only 2 percent of all mined diamonds, and many diamonds of that type are too low in quality for treatment to be worthwhile, the vast majority of mined diamonds must not be treated to have their color removed. However, we are not told what proportion of all mined diamonds are naturally colorless. Naturally colorless diamonds may be far rarer even than the uncommon diamonds that have been treated to have their color removed. Thus, for all we can tell from the passage, it could well be that most colorless diamonds sold by jewelers have been treated to remove all color. A. Even if some types of diamonds command higher prices for uses other than as gemstones, the types discussed in the passage evidently command high enough prices as gemstones to be sold as such by jewelers. B. Correct. The argument does not work if naturally colorless diamonds are rarer than treated diamonds, as they may be for all we can tell from the information provided. C. The argument's conclusion is only that jewelers sell more naturally colorless diamonds than diamonds treated to be colorless. Whether jewelers sell any colored diamonds or other gemstones is irrelevant. D. The argument only concerns the types of colorless diamonds sold now, not the types that may be sold in the future if other treatment methods are discovered. E. The argument does suggest this difficulty but implies that even so there are too few treated diamonds available for jewelers to sell in place of naturally colorless ones. The correct answer is B.
19. Jay: Of course there are many good reasons to support the expansion of preventive medical care, but arguments claiming that it will lead to greater societal economic gains are misguided. Some of the greatest societal expenses arise from frequent urgent-care needs for people who have attained a long life due to preventive care. Sunil: Your argument fails because you neglect economic gains outside the health care system: society suffers an economic loss when any of its productive members suffer preventable illnesses. Sunil's response to Jay makes which of the following assumptions?
A.Those who receive preventive care are not more likely to need urgent care than are those who do not receive preventive care.
B.Jay intends the phrase "economic gains" to refer only to gains accruing to institutions within the health care system.
C.Productive members of society are more likely than others to suffer preventable illnesses.
D.The economic contributions of those who receive preventive medical care may outweigh the economic losses caused by preventive care.
E.Jay is incorrect in stating that patients who receive preventive medical care are long-lived.
A B C D E
D
[解析] Argument Construction Situation Some of the greatest societal expenses arise from frequent urgent-care needs for people who have reached old age thanks to preventive medical care. But society also suffers economic loss when any of its productive members suffer preventable illnesses. Reasoning What is Sunil assuming in his argument that Jay's argument fails? Jay implies that by helping people live longer, expanding preventive medical care may actually increase the amount of urgent medical care people need over the course of their lives, and that societal expenses for this additional urgent care may equal or exceed any societal economic benefits from expanding preventive care. Sunil responds by implying that expanding preventive care would allow society to avoid economic losses from lost productivity caused by preventable illnesses. In order for Sunil's argument to establish that Jay's argument fails, the potential economic benefits that Sunil implies would arise from expanded preventive care must be greater than the economic losses from the increased need for urgent care that Jay points out. A. This is not an assumption that underpins Sunil's suggestion that the societal economic benefits from expanded preventive care may exceed any resulting economic losses from urgent care. B. If Jay intends the phrase "economic gains" to refer only to gains within the health care system, then Sunil's point about economic gains outside the health care system is not even relevant to Jay's argument about economic gains within it. C. Even if productive members of society are not more likely than others to suffer preventable illnesses, it still may be true, as Sunil suggests, that the economic benefits of preventing productive members of society from suffering those illnesses may outweigh the economic losses of doing so. In that case, Jay's argument could still fail in the way Sunil indicates. D. Correct. Sunil must assume this in order to rebut Jay's argument. As explained above, if the economic contributions of those receiving preventive care definitely do not outweigh the economic losses caused by preventive care, then Sunil's implicit point that expanding preventive care would help to prevent the loss of such contributions is insufficient to rebut Jay's argument. E. Whether Jay is correct or incorrect in this respect, Sunil may be correct that Jay's argument fails because Jay has neglected to consider how preventive care produces larger economic gains outside the health care system. The correct answer is D.
20. Boreal owls range over a much larger area than do other owls of similar size. The reason for this behavior is probably that the small mammals on which owls feed are especially scarce in the forests where boreal owls live, and the relative scarcity of prey requires the owls to range more extensively to find sufficient food. Which of the following, if true, most helps to confirm the explanation above?
A.Some boreal owls range over an area eight times larger than the area over which any other owl of similar size ranges.
B.Boreal owls range over larger areas in regions where food of the sort eaten by small mammals is sparse than they do in regions where such food is abundant.
C.After their young hatch, boreal owls must hunt more often than before in order to feed both themselves and their newly hatched young.
D.Sometimes individual boreal owls hunt near a single location for many weeks at a time and do not range farther than a few hundred yards.
E.The boreal owl requires less food, relative to its weight, than is required by members of other owl species,
A B C D E
B
[解析] Argument Evaluation Situation The small mammals on which owls prey are relatively scarce in the forests where boreal owls live. That is why boreal owls range more extensively than do other, similarly sized owls in search of food. Reasoning Which choice, if true, would most help confirm the proposed explanation? One way to confirm an explanation is by finding further information that one would expect to be true if the explanation is valid. If the explanation in the passage is valid, then one would expect that variations in the population density of available small-animal prey for boreal owls would be accompanied by variations in the ranges of the boreal owls. Naturally the population density of available small-animal prey is likely to be affected by how plentiful food is for those small animals. A. The comparison between different groups of boreal owls is not relevant to the comparison between boreal owls and other owls. B. Correct. This indicates that abundance of food for the boreal owls' small-animal prey in an area (and therefore abundance of small animals in that area) correlates with a smaller range for the boreal owls there. This strengthens the proposed explanation. C. This option concerns a correlation between owls' need for food and the frequency with which owls hunt, whereas the phenomenon described in the passage and the proposed explanation have to do with the range over which owls hunt. D. If one were to assume that boreal owls never hunt near a single location for weeks, that would in no way undermine the proposed explanation. E. If anything, this option tends to undermine the proposed explanation, because it suggests the possibility that boreal owls need not make up for the relative scarcity of prey in their habitats by ranging over larger areas. The correct answer is B.
21. Microbiologist: A lethal strain of salmonella recently showed up in a European country, causing an outbreak of illness that killed two people and infected twenty-seven others. Investigators blame the severity of the outbreak on the overuse of antibiotics, since the salmonella bacteria tested were shown to be drug-resistant. But this is unlikely because patients in the country where the outbreak occurred cannot obtain antibiotics to treat illness without a prescription, and the country's doctors prescribe antibiotics less readily than do doctors in any other European country. Which of the following, if true, would most weaken the microbiologist's reasoning?
A.Physicians in the country where the outbreak occurred have become hesitant to prescribe antibiotics since they are frequently in short supply.
B.People in the country where the outbreak occurred often consume foods produced from animals that eat antibiotics-laden livestock feed.
C.Use of antibiotics in two countries that neighbor the country where the outbreak occurred has risen over the past decade.
D.Drug-resistant strains of salmonella have not been found in countries in which antibiotics are not generally available.
E.Salmonella has been shown to spread easily along the distribution chains of certain vegetables, such as raw tomatoes.
A B C D E
B
[解析] Argument Evaluation Situation Antibiotic-resistant salmonella caused an outbreak of illness in a European country where patients need prescriptions to obtain antibiotics and where doctors dispense such prescriptions less readily than in other European countries. Reasoning What evidence would most strongly suggest that overuse of antibiotics was likely responsible for the outbreak, despite the cited facts? The microbiologist reasons that because patients need prescriptions to obtain antibiotics in the country where the outbreak occurred, and the country's doctors dispense such prescriptions less readily than doctors in other European countries do, antibiotics are probably not being overused in the country—so antibiotic overuse was probably not responsible for the outbreak. Implicit in the microbiologist's reasoning is the assumption that overuse of antibiotics, if it had occurred, could probably have resulted only from overprescribing of antibiotics by physicians to treat illness in people in the country in question. Any evidence casting doubt on this complex assumption would suggest a weakness in the microbiologist's reasoning. A. This strengthens the argument by providing additional evidence that antibiotics are not being overprescribed in the country. B. Correct. This weakens the microbiologist's argument by indicating that an assumption implicit in the argument may be false: the salmonella outbreak could easily by explained by overuse of antibiotics in livestock feed (perhaps imported from other countries). C. Even if antibiotic use has risen in the two neighboring countries, antibiotics still might be underused in both countries. D. This suggests that antibiotic-resistant salmonella arises only in countries where antibiotics are used; even if this were true it would be quite compatible with the microbiologist's argument and does not weaken that argument. E. This describes one mechanism by which salmonella can spread in a population; it says nothing about whether an outbreak of antibiotic-resistant strains of salmonella might have been caused by antibiotic overuse. The correct answer is B.
22. For over two centuries, no one had been able to make Damascus blades—blades with a distinctive serpentine surface pattern—but a contemporary sword maker may just have rediscovered how. Using iron with trace impurities that precisely matched those present in the iron used in historic Damascus blades, this contemporary sword maker seems to have finally hit on an intricate process by which he can produce a blade indistinguishable from a true Damascus blade. Which of the following, if true, provides the strongest support for the hypothesis that trace impurities in the iron are essential for the production of Damascus blades?
A.There are surface features of every Damascus blade—including the blades produced by the contemporary sword maker—that are unique to that blade.
B.The iron with which the contemporary sword maker made Damascus blades came from a source of iron that was unknown two centuries ago.
C.Almost all the tools used by the contemporary sword maker were updated versions of tools that were used by sword makers over two centuries ago.
D.Production of Damascus blades by sword makers of the past ceased abruptly after those sword makers' original source of iron became exhausted.
E.Although Damascus blades were renowned for maintaining a sharp edge, the blade made by the contemporary sword maker suggests that they may have maintained their edge less well than blades made using what is now the standard process for making blades.
A B C D E
D
[解析] Argument Evaluation Situation A sword maker may have recently rediscovered how to make Damascus blades using iron with trace impurities matching those in the iron from which historic Damascus blades were wrought. Reasoning What evidence would suggest that the trace impurities are essential for producing Damascus blades? The passage says the sword maker seems to have created blades indistinguishable from historic Damascus blades by using iron with the same trace impurities found in those blades. But that does not prove the trace impurities are essential to the process. Evidence suggesting that Damascus blades have never been made from iron without the trace impurities would support the hypothesis that the trace impurities are essential to their manufacture. A. Damascus blades could vary in their surface features whether or not trace impurities are essential for their manufacture. B. Whatever the source of the iron the contemporary sword maker used, it contains the same trace impurities as the iron historically used to make Damascus blades, which is what the hypothesis is about. C. If anything, this might cast doubt on the hypothesis by suggesting that the special tools rather than the trace impurities could account for the distinctive features of Damascus blades. D. Correct. This suggests that when the historic sword makers lost access to the special iron with its trace impurities, they could no longer make Damascus blades. THUS, it supports the hypothesis that the trace impurities are necessary for manufacturing Damascus blades. E. Even if Damascus blades maintained their edges less well than most contemporary blades do, the trace impurities may not have been essential for manufacturing them. The correct answer is D.
23. Images from ground-based telescopes are invariably distorted by the Earth's atmosphere. Orbiting space telescopes, however, operating above Earth's atmosphere, should provide superbly detailed images. Therefore, ground-based telescopes will soon become obsolete for advanced astronomical research purposes. Which of the following statements, if true, would cast the most doubt on the conclusion drawn above?
A.An orbiting space telescope due to be launched this year is far behind schedule and over budget, whereas the largest ground-based telescope was both within budget and on schedule.
B.Ground-based telescopes located on mountain summits are not subject to the kinds of atmospheric distortion which, at low altitudes, make stars appear to twinkle.
C.By careful choice of observatory location, it is possible for large-aperture telescopes to avoid most of the kind of wind turbulence that can distort image quality.
D.When large-aperture telescopes are located at high altitudes near the equator, they permit the best Earth-based observations of the center of the Milky Way Galaxy, a prime target of astronomical research.
E.Detailed spectral analyses, upon which astronomers rely for determining the chemical composition and evolutionary history of stars, require telescopes with more light-gathering capacity than space telescopes can provide.
A B C D E
E
[解析] Argument Evaluation Situation Earth's atmosphere distorts images from ground-based telescopes, whereas space telescopes orbiting above the atmosphere should provide superbly detailed images. Reasoning What evidence would undermine the claim that ground-based telescopes will soon become obsolete for advanced astronomical research? The argument implicitly assumes that advanced astronomical research can be accomplished more effectively with the more detailed, less distorted images produced by space telescopes and that therefore almost all advanced astronomical research will soon be conducted with space telescopes. This reasoning would be undermined by evidence that ground-based telescopes have substantial advantages for advanced astronomical research despite their distorted images or by evidence that space telescopes will not soon become common or affordable enough to support most advanced astronomical research. A. Even if this is true, there may be several orbiting space telescopes that will be, or have been, launched on schedule and within budget, so this option does not cast doubt on the conclusion of the argument. B. Ground-based telescopes on mountain summits are still subject to more atmospheric distortion than are space telescopes orbiting above the atmosphere. C. Atmospheric distortion of telescopic images may result mainly from factors other than wind turbulence. D. Even the best Earth-based observations of the center of the Milky Way Galaxy may be vastly inferior to space-based observations. E. Correct. This indicates an inherent limitation of space-based telescopes: unlike Earth-based telescopes, they lack the light-gathering capacity that astronomers need to perform one of their primary tasks, i.e., detailed spectral analyses. So Earth-based telescopes are unlikely to soon become obsolete. The correct answer is E.
24. Generally scientists enter their field with the goal of doing important new research and accept as their colleagues those with similar motivation. Therefore, when any scientist wins renown as an expounder of science to general audiences, most other scientists conclude that this popularizer should no longer be regarded as a true colleague. The explanation offered above for the low esteem in which scientific popularizers fire held by research scientists assumes that
A.serious scientific research is not a solitary activity, but relies on active cooperation among a group of colleagues
B.research scientists tend not to regard as colleagues those scientists whose renown they envy
C.a scientist can become a famous popularizer without having completed any important research
D.research scientists believe that those who are well known as popularizers of science are not motivated to do important new research
E.no important new research can be accessible to or accurately assessed by those who are not themselves scientists
A B C D E
D
[解析] Argument Construction Situation Research scientists desire to do important new research and treat as colleagues just those who have a similar desire. When a scientist becomes popular among a general audience for explaining principles of science, other scientists have less esteem for this popularizer, no longer regarding such a scientist as a serious colleague. Reasoning What assumption do research scientists make about scientists who become popularizers? The community of scientists shares a common goal: to do important new research. What would cause this community to disapprove of a popularizer and to cease to regard the popularizer as a colleague? It must be because many scientists believe that becoming a popularizer is incompatible with desiring to do important new research. A. Many scientists make this assumption, of course—but it is not an assumption on which the explanation specifically depends. The explanation concerns the scientists' motivation, not their style of doing research. B. This statement gives another reason that scientists may reject a popularizer, but because it is not the reason implied in the passage, it is not assumed. C. Even if this is true, it does not address the core issue of the argument: what scientists believe about the motivation of popularizers. D. Correct. This statement properly identifies an assumption on which the explanation for scientists' rejection of popularizers depends. E. The passage is not concerned with whether nonscientists can understand new research, but rather with the beliefs and motivations of scientists who reject popularizers as colleagues. The correct answer is D.
25. Urban planner: When a city loses population due to migration, property taxes in that city tend to rise. This is because there are then fewer residents paying to maintain an infrastructure that was designed to support more people. Rising property taxes, in turn, drive more residents away, compounding the problem. Since the city of Stonebridge is starting to lose population, the city government should therefore refrain from raising property taxes. Which of the following, if true, would most weaken the urban planner's argument?
A.If Stonebridge does not raise taxes on its residents to maintain its infrastructure, the city will become much less attractive to live in as that infrastructure decays.
B.Stonebridge at present benefits from grants provided by the national government to help maintain certain parts of its infrastructure.
C.If there is a small increase in property taxes in Stonebridge and a slightly larger proportion of total revenue than at present is allocated to infrastructure maintenance, the funding will be adequate for that purpose.
D.Demographers project that the population of a region that includes Stonebridge will start to increase substantially within the next several years.
E.The property taxes in Stonebridge are significantly lower than those in many larger cities.
A B C D E
A
[解析] Argument Evaluation Situation When a city loses population due to migration, fewer residents remain to pay to maintain the city's infrastructure, so property taxes tend to rise. These rising property taxes then drive even more residents away. The city of Stonebridge is starting to lose population, so Stonebridge's government should not raise property taxes. Reasoning What would weaken the urban planner's justification for concluding that Stonebridge's government should refrain from raising property taxes? The urban planner implicitly reasons that raising property taxes in Stonebridge in order to maintain the city's infrastructure would make the city lose even more residents, leaving even fewer paying to maintain the infrastructure, and that this would worsen the funding problem the tax increase would have been intended to solve. The urban planner's argument would be weakened by any evidence that raising property taxes in Stonebridge would not drive residents away or that refraining from raising property taxes would cause the same problems as raising them would cause, or worse. A. Correct. This suggests that refraining from raising property taxes could drive more residents out of Stonebridge than raising them would, and thus would not help the city avoid the problem the urban planner describes. B. This does slightly weaken the argument because the grants may still be provided to maintain certain parts of the infrastructure, even if increased property taxes drive more residents away. But losing more residents could still make it harder to raise enough funds to maintain the rest of the city's infrastructure, as the urban planner argues. C. Even if this approach would address the immediate maintenance funding problem, the small increase in property taxes could still drive more residents away, forcing additional future tax increases on those who remain, just as the urban planner suggests. D. This does slightly weaken the argument, but the residents who will move to the region might still avoid moving to Stonebridge if the property taxes there are too high, and those who live in Stonebridge might still move to other cities in the region. E. Residents fleeing Stonebridge because of high property taxes would likely avoid moving to the many larger cities with even higher property taxes, but they might be happy to move to many other places with low property taxes. The correct answer is A.
26. Which of the following most logically completes the argument? Utrania was formerly a major petroleum exporter, but in recent decades economic stagnation and restrictive regulations inhibited investment in new oil fields. In consequence, Utranian oil exports dropped steadily as old fields became depleted. Utrania's currently improving economic situation, together with less-restrictive regulations, will undoubtedly result in the rapid development of new fields. However it would be premature to conclude that the rapid development of new fields will result in higher oil exports, because ______.
A.the price of oil is expected to remain relatively stable over the next several years
B.the improvement in the economic situation in Utrania is expected to result in a dramatic increase in the proportion of Utranians who own automobiles
C.most of the investment in new oil fields in Utrania is expected to come from foreign sources
D.new technology is available to recover oil from old oil fields formerly regarded as depleted
E.many of the new oil fields in Utrania are likely to be as productive as those that were developed during the period when Utrania was a major oil exporter
A B C D E
B
[解析] Argument Construction Situation A country that had been a major oil exporter has seen its exports decline in recent decades due to economic stagnation, a failure to invest in new fields, and the steady depletion of its old fields. But looser regulations and an improving economy will bring rapid development of new oil fields in the country. Reasoning Which of the options would most logically complete the argument? The passage describes the conditions that led to Utrania's no longer being a major oil exporter: a lack of investment in new oil fields due to a stagnant economy and restrictive regulations. The passage then says that due to changed regulatory and economic conditions, there will now be rapid development of new oil fields. Nonetheless, this might not bring about an increase in Utrania's oil exports. To logically complete the argument, one must explain how oil exports might not increase even when the condition that led to decreased oil exports has been removed. Suppose there were an increase in domestic oil consumption. A dramatic increase in the rate of car ownership in Utrania could reasonably be expected to significantly increase domestic oil consumption, which could eat up the added oil production from the new fields. A. This choice is incorrect. There is no reason why stable oil prices should prevent Utrania's oil exports from increasing. B. Correct. An increase in car ownership would increase Utrania's oil consumption—and this supports the claim that oil exports might not increase. C. If anything, this suggests that oil exports should increase. So it would not be a good choice for completion of the argument. D. The advent of new technology allowing oil to be extracted from fields previously thought to be depleted would mean that there is even more reason to think that Utrania's oil exports will increase. E. This does not help to explain why exports would not increase. On the contrary, it suggests that the new fields will lead to increased exports. The correct answer is B.
27. The use of growth-promoting antibiotics in hog farming can weaken their effectiveness in treating humans because such use can spread resistance to those antibiotics among microorganisms. But now the Smee Company, one of the largest pork marketers, may stop buying pork raised on feed containing these antibiotics. Smee has 60 percent of the pork market, and farmers who sell to Smee would certainly stop using antibiotics in order to avoid jeopardizing their sales. So if Smee makes this change, it will probably significantly slow the decline in antibiotics' effectiveness for humans. Which of the following, if true, would most strengthen the argument above?
A.Other major pork marketers will probably stop buying pork raised on feed containing growth-promoting antibiotics if Smee no longer buys such pork.
B.The decline in hog growth due to discontinuation of antibiotics can be offset by improved hygiene.
C.Authorities are promoting the use of antibiotics to which microorganisms have not yet developed resistance.
D.A phaseout of use of antibiotics for hogs in one country reduced usage by over 50 percent over five years.
E.If Smee stops buying pork raised with antibiotics, the firm's costs will probably increase.
A B C D E
A
[解析] Argument Evaluation Situation Using growth-promoting antibiotics in hog farming can produce widespread resistance to antibiotics among microorganisms, thereby making the antibiotics less effective in treating humans. The Smee Company, a pork marketer with 60 percent of the pork market, may stop buying pork raised on feed containing these antibiotics. Reasoning What additional evidence would most help to support the conclusion that if Smee makes the change, it will significantly slow the decline in antibiotics' effectiveness for humans? We are already informed that if Smee makes the change, it will eliminate the use of antibiotics in hog feed by farmers supplying at least 60 percent of the pork market. The argument would be strengthened by evidence that Smee's decision would indirectly cause use of the antibiotics to stop more broadly, for example in hog farms supplying significantly more than 60 percent of the total amount of pork marketed. A. Correct. This suggests that if Smee makes the change, hog farmers supplying other major pork marketers will also have to stop using antibiotics in hog feed, making the change more widespread and thus probably more effective. B. Even if the decline in hog growth from discontinuing the antibiotics cannot be offset, many hog farmers will still have to stop using the antibiotics as a result of Smee's decision. On the other hand, even if the decline can be offset with improved hygiene, that change might be too expensive or difficult to be worth its benefits for most hog farmers. C. Whatever new antibiotics authorities are promoting, microorganisms may soon develop resistance to them as well. Smee may or may not refuse to buy pork raised on feed containing these new antibiotics. D. This is evidence that Smee's decision may significantly reduce antibiotic use in hogs, but it provides no evidence of how this reduction may affect antibiotics' effectiveness for humans. E. If anything, this provides reason to suspect that Smee will not stick with the change for long after the costs increase, so it weakens rather than strengthens the argument that the change will significantly slow the decline in antibiotics' effectiveness. The correct answer is A.
28. In an experiment, volunteers walked individually through a dark, abandoned theater. Half of the volunteers had been told that the theater was haunted and the other half that it was under renovation. The first half reported significantly more unusual experiences than the second did. The researchers concluded that reports of encounters with ghosts and other supernatural entities generally result from prior expectations of such experiences. Which of the following, if true, would most seriously weaken the researchers' reasoning?
A.None of the volunteers in the second half believed that the unusual experiences they reported were supernatural.
B.All of the volunteers in the first half believed that the researchers' statement that the theater was haunted was a lie.
C.Before being told about the theater, the volunteers within each group varied considerably in their prior beliefs about supernatural experiences.
D.Each unusual experience reported by the volunteers had a cause that did not involve the supernatural.
E.The researchers did not believe that the theater was haunted.
A B C D E
B
[解析] Argument Evaluation Situation Volunteers in an experiment walked through a dark, abandoned theater. Those who had been told the theater was haunted reported more unusual experiences than those who had been told it was under renovation. Reasoning What evidence would most strongly suggest that the experimental results do not indicate that reports of supernatural encounters result from prior expectations of such experiences? The researcher assumes that the half of the volunteers who had been told the theater was haunted were more inclined to expect supernatural experiences in the theater than were the other half of the volunteers. Based on this assumption and the greater incidence of reports of unusual experiences among the first half of the volunteers, the researcher concludes that prior expectation of supernatural experiences makes people more likely to report such experiences. The researchers' reasoning would be weakened by evidence that the volunteers did not actually have the expectations the researchers assumed them to have, or by evidence that any such expectations did not influence their reports. A. This strengthens the argument by indicating that the volunteers whom the researchers did not lead to expect supernatural experiences reported no such experiences. B. Correct. If none of the volunteers believed the researchers' claim that the theater was haunted, then the implicit assumption that several of those volunteers expected supernatural experiences in the theater is flawed, and so the inference that their prior expectations probably account for their reports of supernatural experiences is flawed. C. This is compatible with the researchers' inference and does not undermine it. Even if the volunteers' initial beliefs about supernatural experiences varied, the researchers' claims about the theater might have strongly influenced how many volunteers in each group expected to have such experiences in the theater specifically. D. The researchers argue that the volunteers' prior expectations account for all the reports of unusual experiences, and this is compatible with there being no genuine supernatural occurrences in the theater. E. Whatever the researchers personally believed about the theater, they might still have successfully influenced the volunteers' beliefs about it. The correct answer is B.
29. In order to reduce dependence on imported oil, the government of Jalica has imposed minimum fuel-efficiency requirements on all new cars, beginning this year. The more fuel-efficient a car, the less pollution it produces per mile driven. As Jalicans replace their old cars with cars that meet the new requirements, annual pollution from car traffic is likely to decrease in Jalica. Which of the following, if true, most seriously weakens the argument?
A.In Jalica, domestically produced oil is more expensive than imported oil.
B.The Jalican government did not intend the new fuel-efficiency requirement to be a pollution-reduction measure.
C.Some pollution-control devices mandated in Jalica make cars less fuel-efficient than they would be without those devices.
D.The new regulation requires no change in the chemical formulation of fuel for cars in Jalica.
E.Jalicans who get cars that are more fuel-efficient tend to do more driving than before.
A B C D E
E
[解析] Argument Evaluation Situation The Jalican government is requiring all new cars to meet minimum fuel-efficiency requirements starting this year. Cars that are more fuel efficient produce less pollution per mile driven. Reasoning What evidence would suggest that annual pollution from car traffic will not decrease in Jalica, despite the new policy? Air pollution from car traffic is unlikely to decrease if the new standards will result in more cars on the road or more miles driven per car; or if air pollution from car traffic in Jalica is increasing because of unrelated factors such as growing numbers of Jalicans who can afford cars, construction of more roads, etc. Evidence that any of these factors is present would cast doubt on the argument's conclusion and thus weaken the argument. A. The question at issue is not whether the new policy will reduce dependence on imported oil as the government intends, but rather whether it will reduce air pollution from car traffic. B. A government policy may have consequences that the government did not intend it to have. C. Even if these pollution-control devices make cars less fuel efficient, the new fuel-efficiency standards may still improve cars' average fuel efficiency and thereby reduce air pollution. D. Even if the fuel is unchanged, the new fuel-efficiency standards may still result in cars using less fuel and may thereby reduce air pollution. E. Correct. If the new fuel-efficient cars are driven more miles per year than older cars are, they may produce as much or more pollution per year than older cars do even though they produce less pollution per mile driven. The correct answer is E.
30. Plantings of cotton bioengineered to produce its own insecticide against bollworms, a major cause of crop failure, sustained little bollworm damage until this year. This year the plantings are being seriously damaged by bollworms. Bollworms, however, are not necessarily developing resistance to the cotton's insecticide. Bollworms breed on corn, and last year more corn than usual was planted throughout cotton-growing regions. So it is likely that the cotton is simply being overwhelmed by corn-bred bollworms. In evaluating the argument, which of the following would it be most useful to establish?
A.Whether corn could be bioengineered to produce the insecticide
B.Whether plantings of cotton that does not produce the insecticide are suffering unusually extensive damage from bollworms this year
C.Whether other crops that have been bioengineered to produce their own insecticide successfully resist the pests against which the insecticide was to protect them
D.Whether plantings of bioengineered cotton are frequently damaged by insect pests other than bollworms
E.Whether there are insecticides that can be used against bollworms that have developed resistance to the insecticide produced by the bioengineered cotton
A B C D E
B
[解析] Argument Evaluation Situation Although plantings of cotton bioengineered to produce an insecticide to combat bollworms were little damaged by the pests in previous years, they are being severely damaged this year. Since the bollworms breed on corn, and there has been more corn planted this year in cotton-growing areas, the cotton is probably being overwhelmed by the corn-bred bollworms. Reasoning In evaluating the argument, which question would it be most useful to have answered? The argument states that the bioengineered cotton crop failures this year (1) have likely been due to the increased corn plantings and (2) not due to the pests having developed a resistance to the insecticide. This also implies (3) that the failures are not due to some third factor. It would be useful to know how the bioengineered cotton is faring in comparison to the rest of this year's cotton crop. If the bioengineered cotton is faring better against the bollworms, that fact would support the argument because it would suggest that the insecticide is still combating bollworms. If, on the other hand, the bioengineered cotton is being more severely ravaged by bollworms than is other cotton, that suggests that there is some third cause that is primarily at fault. A. This would probably be useful information to those trying to alleviate the bollworm problem in bioengineered cotton. But whether such corn could be developed has no bearing on what is causing the bioengineered cotton to be damaged by bollworms this year. B. Correct. If bollworm damage on non-bioengineered cotton is worse than usual this year, then bollworm infestation in general is simply worse than usual, so pesticide resistance does not need to be invoked to explain the bollworm attacks on the bioengineered cotton. C. Even if other crops that have been bioengineered to resist pests have not successfully resisted them, that fact would not mean that the same is true of this cotton. Furthermore, the facts already suggest that the bioengineered cotton has resisted bollworms. D. Whether other types of pests often damage bioengineered cotton has no bearing on why bollworms are damaging this type of cotton more this year than in the past. E. This, too, might be useful information to those trying to alleviate the bollworm problem in bioengineered cotton, but it is not particularly useful in evaluating the argument. Even if there are pesticides that could be used against bollworms that have developed resistance to the insecticide of the bioengineered cotton, that does not mean that such pesticides are being used this year. The correct answer is B.
31. Typically during thunderstorms most lightning strikes carry a negative electric charge; only a few carry a positive charge. Thunderstorms with unusually high proportions of positive-charge strikes tend to occur in smoky areas near forest fires. The fact that smoke carries positively charged smoke particles into the air above a fire suggests the hypothesis that the extra positive strikes occur because of the presence of such particles in the storm clouds. Which of the following, if discovered to be true, most seriously undermines the hypothesis?
A.Other kinds of rare lightning also occur with unusually high frequency in the vicinity of forest fires.
B.The positive-charge strikes that occur near forest fires tend to be no more powerful than positive strikes normally are.
C.A positive-charge strike is as likely to start a forest fire as a negative-charge strike is.
D.Thunderstorms that occur in drifting clouds of smoke have extra positive-charge strikes weeks after the charge of the smoke particles has dissipated.
E.The total number of lightning strikes during a thunderstorm is usually within the normal range in the vicinity of a forest fire.
A B C D E
D
[解析] Argument Evaluation Situation Thunderstorms with unusually high proportions of positive-charge lightning strikes tend to occur in smoky areas near forest fires. Smoke carries positively charged particles into the air above fires, suggesting that smoke particles in storm clouds are responsible for the higher proportion of positive strikes. Reasoning What would cast doubt on the hypothesis that the extra positive-charge lightning strikes in thunderstorms near forest fires result from positively charged smoke particles carried into the storm clouds? The hypothesis would be weakened by evidence that the positively charged smoke particles do not enter the storm clouds in the first place, or that they do not retain their charge in the clouds long enough to produce an effect, or that their positive charge cannot affect the charges of the storm's lightning strikes in any case, or that some other factor tends to make the lightning strikes above these storms positively charged. A. It could be that positively charged smoke particles cause these other kinds of rare lightning, too, so this does not seriously undermine the hypothesis. B. The hypothesis is not about the power of the positive-charge lightning strikes, only about why a high proportion of them occur in thunderstorms near forest fires. C. The hypothesis is about why positive-charge strikes tend to occur in smoky areas near forest fires that have already started before the strikes occur. Furthermore, an equal likelihood of positive-charge and negative-charge strikes starting fires cannot explain a correlation between fires and positive-charge strikes specifically. D. Correct. This means that even when drifting clouds of smoke persist for weeks after a fire, when the charge of their particles has already dissipated, the smoke somehow still makes the strikes positively charged in any thunderstorms arising within it. If so, some factor other than positively charged smoke particles must affect the strikes' charge. E. This information does not undermine the hypothesis. The hypothesis does not concern the possibility that there might be more lightning strikes in the vicinity of forest fires; rather it concerns the proportion of all such lightning strikes that are positively charged. The correct answer is D.
32. Many gardeners believe that the variety of clematis vine that is most popular among gardeners in North America is jackmanii. This belief is apparently correct since, of the one million clematis plants sold per year by the largest clematis nursery in North America, ten percent are jackmanii. Which of the following is an assumption on which the argument depends?
A.The nursery sells more than ten different varieties of clematis.
B.The largest clematis nursery in North America sells nothing but clematis plants.
C.Some of the jackmanii sold by the nursery are sold to gardeners outside North America.
D.Most North American gardeners grow clematis in their gardens.
E.For all nurseries in North America that specialize in clematis, at least ten percent of the clematis plants they sell are jackmanii.
A B C D E
A
[解析] Argument Construction Situation Of the clematis plants sold by the largest clematis nursery in North America, 10 percent are jackmanii, which many gardeners believe to be the most popular variety of clematis in North America. Reasoning What must be true in order for the fact that l O percent of the clematis sold at the nursery are jackmanii to provide evidence that jackmanii is the most popular variety of clematis in North America? The argument assumes that sales of different varieties of clematis at the nursery reflect the relative levels of popularity of those varieties among North American gardeners. It also assumes that jackmanii is the best-selling clematis variety at the nursery, an assumption which requires that less than 10 percent of the nursery's clematis sales are of any one variety other than jackmanii. A. Correct. Suppose the nursery sold ten or fewer varieties of clematis. Then at least one variety other than jackmanii would have to account for at least 10 percent of the nursery's clematis sales, so jackmanii would not be the best-selling clematis variety at the nursery as the argument assumes. B. The argument only concerns how popular jackmanii is relative to other varieties of clematis, not relative to any plants other than clematis that the nursery may sell. C. If anything, this would weaken the argument by suggesting that the nursery's jackmanii sales might reflect jackmanii's popularity outside North America more than its popularity within North America. D. This would indicate that clematis is a popular plant among North American gardeners, not that jackmanii is the most popular variety of clematis. E. Even if jackmanii accounts for less than 10 percent of clematis sales at a few individual nurseries, it may still account for 10 percent or more of North American clematis sales overall. The correct answer is A.
33. Since 1990 the percentage of bacterial sinus infections in Aqadestan that are resistant to the antibiotic perxicillin has increased substantially. Bacteria can quickly develop resistance to an antibiotic when it is prescribed indiscriminately or when patients fail to take it as prescribed. Since perxicillin has not been indiscriminately prescribed, health officials hypothesize that the increase in perxicillin-resistant sinus infections is largely due to patients' failure to take this medication as prescribed. Which of the following, if true of Aqadestan, provides most support for the health officials' hypothesis?
A.Resistance to several other commonly prescribed antibiotics has not increased since 1990 in Aqadestan.
B.A large number of Aqadestanis never seek medical help when they have a sinus infection.
C.When it first became available, perxicillin was much more effective in treating bacterial sinus infections than any other antibiotic used for such infections at the time.
D.Many patients who take perxicillin experience severe side effects within the first few days of their prescribed regimen.
E.Aqadestani health clinics provide antibiotics to their patients at cost.
A B C D E
D
[解析] Argument Construction Situation In Aqadestan the percentage of bacterial sinus infections resistant to the antibiotic perxicillin has been increasing even though perxicillin has not been indiscriminately prescribed. Reasoning What evidence most strongly suggests that the main reason perxicillin-resistant sinus infections are becoming more common is that patients are failing to take perxicillin as prescribed? Any evidence suggesting that patients have in fact been failing to take perxicillin as prescribed would support the hypothesis, as would any evidence casting doubt on other possible explanations for the increasing proportion of perxicillin-resistant sinus infections. A. This suggests that some factor specific to perxicillin is increasing bacterial resistance to it, but that could be true whether or not the factor is patients' failure to take perxicillin as prescribed. B. If anything, this weakens the argument by suggesting that most people with sinus infections are never prescribed perxicillin, and that therefore relatively few people are getting prescriptions and then failing to follow them. C. The relative effectiveness ofperxicillin when it first became available does not suggest that the reason it is now becoming less effective is that many patients are failing to take it as prescribed. D. Correct. These side effects would discourage patients from taking perxicillin as prescribed, so their existence provides evidence that many patients are not taking it as prescribed. E. If the clinics do not charge extra for perxicillin, that would make it more affordable and hence easier for many patients to take as prescribed. The correct answer is D.
34. Psychologist: In a study, researchers gave 100 volunteers a psychological questionnaire designed to measure their self-esteem. The researchers then asked each volunteer to rate the strength of his or her own social skills. The volunteers with the highest levels of self-esteem consistently rated themselves as having much better social skills than did the volunteers with moderate levels. This suggests that attaining an exceptionally high level of self-esteem greatly improves one's social skills. The psychologist's argument is most vulnerable to criticism on which of the following grounds?
A.It fails to adequately address the possibility that many of the volunteers may not have understood what the psychological questionnaire was designed to measure.
B.It takes for granted that the volunteers with the highest levels of self-esteem had better social skills than did the other volunteers, even before the former volunteers had attained their high levels of self-esteem.
C.It overlooks the possibility that people with very high levels of self-esteem may tend to have a less accurate perception of the strength of their own social skills than do people with moderate levels of self-esteem.
D.It relies on evidence from a group of volunteers that is too small to provide any support for any inferences regarding people in general.
E.It overlooks the possibility that factors other than level of self-esteem may be of much greater importance in determining the strength of one's social skills.
A B C D E
C
[解析] Argument Evaluation Situation In a psychological study of 100 volunteers, those found to have the highest self-esteem consistently rated themselves as having much better social skills than did those found to have moderate self-esteem. Reasoning What is wrong with the psychologist citing the study's results to justify the conclusion that exceptionally high self-esteem greatly improves social skills? The psychologist reasons that the study shows a correlation between very high self-esteem and how highly one rates one's social skills, and that this correlation in turn suggests that very high self-esteem improves social skills. This argument is vulnerable to at least two criticisms: First, the argument assumes that the volunteers' ratings of their own social skills are generally accurate. But very high self-esteem might in many cases result from a tendency to overestimate oneself and one's skills, including one's social skills. Second, the argument fails to address the possibility that good social skills promote high self-esteem rather than vice versa, as well as the possibility that some third factor (such as a sunny disposition or fortunate circumstances) promotes both high self-esteem and good social skills. A. An experiment's subjects do not have to understand the experiment's design in order for the experimental results to be accurate. B. To the contrary, the argument concludes that the volunteers with the highest self-esteem attained their enhanced social skills as a result of attaining such high self-esteem. C. Correct. As explained above, very high self-esteem may often result from a tendency to overestimate oneself in general, and thus to overestimate one's social skills. D. A group of 100 volunteers is large enough for an experiment to provide at least a little support for at least some inferences regarding people in general. E. As explained above, the argument overlooks the possibility that some third factor may play a significant role in determining the strength of one's social skills. But even if some factor other than self-esteem is more important in determining the strength of social skills, that would still be compatible with very high self-esteem being of some importance in improving one's social skills. The correct answer is C.
35. A product that represents a clear technological advance over competing products can generally command a high price. Because technological advances tend to be quickly surpassed and companies want to make large profits while they still can, many companies charge the maximum possible price for such a product. But large profits on the new product will give competitors a strong incentive to quickly match the new product's capabilities. Consequently, the strategy to maximize overall profit from a new product is to charge less than the greatest possible price. In the argument above, the two portions in boldface play which of the following roles?
A.The first is a consideration raised to argue that a certain strategy is counterproductive; the second presents that strategy.
B.The first is a consideration raised to support the strategy that the argument recommends; the second presents that strategy.
C.The first is a consideration raised to help explain the popularity of a certain strategy; the second presents that strategy.
D.The first is an assumption, rejected by the argument, that has been used to justify a course of action; the second presents that course of action.
E.The first is a consideration that has been used to justify adopting a certain strategy; the second presents the intended outcome of that strategy.
A B C D E
C
[解析] Argument Construction Situation Often, when a company comes out with an innovative product, it will price the product as high as it can to maximize profits before the competitors quickly catch up. But this is not a good strategy because the very high price of the new product only encourages competitors to match the technological advance more quickly. Reasoning Which option best describes the roles that the boldface portions play in the argument? This type of item concerns only the argument's structure—the way it is intended to work, not the quality of the argument or what might strengthen or weaken the argument. So even if a boldface portion could be used by the argument in a certain way, all that matters is its actual intended role. The fact that technological advances tend to be quickly surpassed serves to partly explain why many companies charge the maximum possible price for such a product. In other words, the first boldface portion helps explain the popularity of the strategy presented in the second boldface portion. The conclusion of the argument, however, is that the strategy exemplified in this latter boldface portion is unwise, so the argument as a whole opposes that strategy. A. Although the first boldface portion could be used as part of an argument that the strategy presented in the second boldface portion is counterproductive, that is not how it is used here. Rather, it immediately follows the word because and serves to explain the occurrence of what is described in the second boldface portion. B. This is clearly wrong because the second boldface portion presents the strategy that the argument opposes. C. Correct. It is the only choice that is consistent with the analysis of the reasoning presented above. D. The first boldface portion is not an assumption rejected by the argument; rather, it is affirmed in the argument. E. The argument does not expressly claim that the first boldface portion has been used to justify the strategy of setting the price as high as possible, although it implies that this is part of the justification that those adopting the strategy would give. More clearly, the second boldface portion does not describe the intended outcome of the strategy, but rather the means of bringing about that intended outcome (maximizing profits, by means of high prices). The correct answer is C.
36. Gortland has long been narrowly self-sufficient in both grain and meat. However, as per capita income in Gortland has risen toward the world average, per capita consumption of meat has also risen toward the world average, and it takes several pounds of grain to produce one pound of meat. Therefore, since per capita income continues to rise, whereas domestic grain production will not increase, Gortland will soon have to import either grain or meat or both. Which of the following is an assumption on which the argument depends?
A.The total acreage devoted to grain production in Gortland will not decrease substantially.
B.The population of Gortland has remained relatively constant during the country's years of growing prosperity.
C.The per capita consumption of meat in Gortland is roughly the same across all income levels.
D.In Gortland, neither meat nor grain is subject to government price controls.
E.People in Gortland who increase their consumption of meat will not radically decrease their consumption of grain.
A B C D E
E
[解析] Argument Construction Situation A country previously self-sufficient in grain and meat will soon have to import one or the other or both because its consumption of meat has risen as per capita income has risen. It takes several pounds of grain to produce one pound of meat. Reasoning What conditions must be true for the conclusion to be true? Meat consumption is rising. What about grain consumption? A sharp reduction in the amount of grain directly consumed by meat eaters could compensate for increased meat consumption, making the conclusion false. If people did radically decrease their grain consumption, it might not be necessary to import grain or meat. Since the argument concludes that the imports are necessary, it assumes that direct consumption of grain by those who begin to eat meat will not plunge. A. The argument makes no assumptions about the acreage devoted to grain; it assumes only that the demand for grain will rise. B. The argument is based on rising per capita income, not population levels. C. The argument involves only meat consumption in general, not its distribution by income level. D. Since the argument does not refer to price controls, it cannot depend on an assumption about them. E. Correct. This statement properly identifies the assumption that those who begin to eat meat do not then greatly decrease their direct consumption of grains. The correct answer is E.
37. To prevent a newly built dam on the Chiff River from blocking the route of fish migrating to breeding grounds upstream, the dam includes a fish pass, a mechanism designed to allow fish through the dam. Before the construction of the dam and fish pass, several thousand fish a day swam upriver during spawning season. But in the first season after the project's completion, only 300 per day made the journey. Clearly, the fish pass is defective. Which of the following, if true, most seriously weakens the argument?
A.Fish that have migrated to the upstream breeding grounds do not return down the Chiff River again.
B.On other rivers in the region, the construction of darns with fish passes has led to only small decreases in the number of fish migrating upstream.
C.The construction of the dam stirred up potentially toxic river sediments that were carried downstream.
D.Populations of migratory fish in the Chiff River have been declining slightly over the last 20 years.
E.During spawning season, the dam releases sufficient water for migratory fish below the dam to swim upstream.
A B C D E
C
[解析] Argument Evaluation Situation A new dam includes a mechanism called a fish pass designed to allow fish to migrate upstream past the dam to their breeding grounds. The number of migrating fish fell from several thousand per day before the dam was built to three hundred per day in the first season after it was built, indicating—according to the argument—that the fish pass is defective. Reasoning What evidence would suggest that the fish pass is not defective? The argument implicitly reasons that a defective fish pass would make it difficult for the fish to migrate, which would explain why the number of migrating fish fell when the dam was completed. Any evidence suggesting an alternative explanation for the reduced number of migrating fish, such as an environmental change that occurred when the dam was built, would cast doubt on the argument's reasoning. A. A defective fish pass could prevent most of the fish from migrating upstream regardless of whether those that succeed ever return downstream. B. This would suggest that dams with properly functioning fish passes do not greatly reduce the number of migrating fish, so it would provide further evidence that the fish pass in this particular dam is defective. C. Correct. This suggests that the toxic sediments may have poisoned the fish and reduced their population. A smaller fish population could be sufficient to explain the reduced number of fish migrating, which casts doubt on the argument's assumption that the explanation for their declining numbers involves the fish pass. D. A slight and gradual ongoing decline in migratory fish populations would not explain an abrupt and extreme decline right after the dam was built. E. This supports the argument's proposed explanation for the declining fish population by ruling out the alternative explanation that the dam does not release enough water for the fish to migrate. The correct answer is C.
38. Music critic: Fewer and fewer musicians are studying classical music, decreasing the likelihood that those with real aptitude for such music will be performing it. Audiences who hear these performances will not appreciate classical music's greatness and will thus decamp to other genres. So to maintain classical music's current meager popularity, we must encourage more young musicians to enter the field. Which of the following, if true, most weakens the music critic's reasoning?
A.Musicians who choose to study classical music do so because they believe they have an aptitude for the music.
B.Classical music's current meager popularity is attributable to the profusion of other genres of music available to listeners.
C.Most people who appreciate classical music come to do so through old recordings rather than live performances.
D.It is possible to enjoy the music in a particular genre even when it is performed by musicians who are not ideally suited for that genre.
E.The continued popularity of a given genre of music depends in part on the audience's being able to understand why that genre attained its original popularity.
A B C D E
C
[解析] Argument Evaluation Situation Fewer musicians are studying classical music. This reduces the likelihood that those performing the music will have real aptitude for it, which in turn reduces audience's appreciation of classical music performances. Reasoning What evidence would cast the most doubt on the support provided for the conclusion that encouraging more young musicians to study classical music is necessary in order to maintain the genre's meager popularity? The music critic's argument is that because fewer talented classical musicians are performing, audiences hearing their performances will fail to appreciate the genre, and thus will abandon it. The critic reasons that to solve this problem, it will be necessary to encourage more young musicians to study classical music so that audiences will eventually be exposed to more talented classical performers and decide the genre is worthwhile after all. The argument would be weakened, for example, by evidence that hearing unremarkable live performances does not really drive many people away from classical music, or that the number of audience members hearing great performances does not depend much on the number of talented performers, or that encouraging young musicians to study classical music is either ineffective or not the only effective way to increase the number of talented classical performers. A. This does not weaken the critic's reasoning. However much confidence musicians studying classical music have in their own talent, a decline in the total number of classical musicians will probably result in a decline in the number of truly talented classical musicians, just as the critic assumes. B. The critic is only proposing a way to at least maintain classical music's current meager popularity, which might be accomplished even if the profusion of other genres prevents classical music's popularity from increasing. C. Correct. This suggests that classical music's meager popularity could at least be maintained by encouraging people to listen to great old recordings of classical music rather than by increasing the supply of great live performances. D. This does weaken the argument slightly. But even if a few audience members manage to enjoy mediocre classical music performances, they might still be more strongly drawn to other genres with more talented performers. E. Listeners exposed to more impressive live performances of classical music by talented performers would probably better understand why classical music was once popular than would listeners exposed only to mediocre classical performances. The correct answer is C.
39. Commemorative plaques cast from brass are a characteristic art form of the Benin culture of West Africa. Some scholars, noting that the oldest surviving plaques date to the 1400s, hypothesize that brass-casting techniques were introduced by the Portuguese, who came to Benin in 1485 A.D. But Portuguese records of that expedition mention cast-brass jewelry sent to Benin's king from neighboring Ife. So it is unlikely that Benin's knowledge of brass casting derived from the Portuguese. Which of the following, if true, most strengthens the argument?
A.The Portuguese records do not indicate whether their expedition of 1485 included metalworkers.
B.The Portuguese had no contact with Ife until the 1500s.
C.In the 1400s the Portuguese did not use cast brass for commemorative plaques.
D.As early as 1500 A.D., Benin artists were making brass plaques incorporating depictions of Europeans.
E.Copper, which is required for making brass, can be found throughout Benin territory.
A B C D E
B
[解析] Argument Construction Situation The oldest surviving cast-brass plaques from the Benin culture date to the 1400s. Records of a Portuguese expedition to Benin in 1485 mention cast-brass jewelry sent to Benin's king from neighboring Ife. Reasoning What additional evidence, when combined with the argument's premises, would most help support the conclusion that Benin's knowledge of brass casting did not derive from the Portuguese? The argument is that since the expedition records indicate that cast-brass jewelry from Ire was already known in Benin when the Portuguese first came there, Benin's knowledge of brass casting probably did not derive from the Portuguese. This argument assumes that receiving the brass-cast jewelry from Ire could have transmitted knowledge of brass casting to Benin, and also that knowledge of brass casting in Ire did not itself derive from the Portuguese. Any evidence supporting either of these assumptions would strengthen the argument. A. This is compatible with a Portuguese origin for brass-casting in Benin. The expedition might well have included metalworkers even if the records do not mention whether it did. Furthermore, other Portuguese expeditions with metalworkers might have quickly followed the initial expedition. B. Correct. If the Portuguese had no contact with Ire before 1500, then Ife's earlier knowledge of brass casting did not derive directly from the Portuguese. This increases the likelihood that knowledge of brass-casting in Benin did not derive from the Portuguese, even if it derived from Ire. C. This is compatible with a Portuguese origin for brass-casting in Benin. Even if the Portuguese did not use cast brass for commemorative plaques, they could have used it for jewelry or other items they brought to Benin or manufactured there, and thus they could have transmitted the knowledge to the Benin culture. D. This leaves open the possibility that the Benin culture learned about brass casting from the Portuguese in 1485 and started using it to produce plaques of this type by 1500. E. Even if copper has always been common in the Benin territory, brass-casting techniques could have been introduced by the Portuguese. The correct answer is B.